Add solutions and graphics to Serway and Jewett v8's chapter 28 problems.
[course.git] / latex / problems / Serway_and_Jewett_8 / problem28.11.tex
1 \begin{problem*}{28.11}
2 A battery with $\EMF=6.00\U{V}$ and no internal resistance supplies
3 current to the circuit shown in Figure~P28.11.  When the double-throw
4 switch $S$ is open as shown in the figure, the current in the battery
5 is $1.00\U{mA}$.  When the switch is closed in position $a$, the
6 current in the battery is $1.20\U{mA}$.  When the switch is closed in
7 position $b$, the current in the battery is $2.00\U{mA}$.  Find the
8 resistances \Part{a} $R_1$, \Part{b} $R_2$, and \Part{c} $R_3$.
9 \begin{center}
10 %  +---R1---+----R2----+
11 %  |        |          |
12 %  |        R2         |
13 %  _        a          |
14 % ___      S ----------+
15 %  |        b          R3
16 %  |        |          |
17 %  +--------+----------+
18 \begin{asy}
19 import Circ;
20
21 MultiTerminal S = switchSPDT(dir=180, label=Label("$S$", align=dir(-70)));
22 label("$a$", S.terminal[2], align=W);
23 label("$b$", S.terminal[1], align=W);
24 MultiTerminal R2v = resistor(S.terminal[2], dir=90, "$R_2$");
25 MultiTerminal R1 = resistor(
26     R2v.terminal[1], dir=180, label=Label("$R_1$", align=N));
27 MultiTerminal R2h = resistor(R2v.terminal[1], "$R_2$");
28 MultiTerminal R3 = resistor(
29     (R2h.terminal[1].x, S.terminal[0].y), dir=-90, "$R_3$");
30 MultiTerminal B = battery(label=Label("$\EMF$", align=W), draw=false);
31 B.centerto(R1.terminal[1], (R1.terminal[1].x, R3.terminal[1].y));
32 B.draw();
33 wire(R1.terminal[1], B.terminal[0]);
34 wire(B.terminal[1], R3.terminal[1], udsq);
35 pair bx = (S.terminal[1].x, R3.terminal[1].y);
36 wire(S.terminal[1], bx);
37 dot(bx);
38 wire(S.terminal[0], R3.terminal[0]);
39 wire(R3.terminal[0], R2h.terminal[1]);
40 dot(R3.terminal[0]);
41 dot(R2v.terminal[1]);
42 \end{asy}
43 \end{center}
44 \end{problem*}
45
46 \begin{solution}
47 When the switch is in position $b$, the vertical $R_2$ resistor
48 recieves no current.  Applying Kirchhoff's loop rule gives
49 \begin{align}
50   0 &= \EMF - I R_3 - I R_2 - I R_1 \\
51   \frac{\EMF}{I} &= R_1 + R_2 + R_3 \;.
52 \end{align}
53
54 When the switch is in position $a$, the two $R_2$ resistors are in
55 parallel, so they can be replaced by an equivalent resistance
56 \begin{equation}
57   R_2' = (1/R_2 + 1/R_2)^{-1} = R_2/2 \;.
58 \end{equation}
59 After you've made this replacement, there is only a single loop in the
60 circuit.  Applying Kirchhoff's loop rule gives
61 \begin{align}
62   0 &= \EMF - I_a R_3 - I_a R_2' - I_a R_1 \\
63   \frac{\EMF}{I_a} &= R_1 + \frac{R_2}{2} + R_3 \;.
64 \end{align}
65
66 When the switch is in position $b$, the vertical $R_2$ resistor
67 recieves no current and $R_3$ is shorted.  Kirchoff's loop rule gives
68 \begin{align}
69   0 &= \EMF - I_b R_2 - I_b R_1 \\
70   \frac{\EMF}{I_b} &= R_1 + R_2 \;.
71 \end{align}
72
73 This gives three equations with three unknowns.  Solve however you
74 like.
75 \begin{align}
76  \begin{pmatrix}
77   \frac{6.00\U{V}}{1.00\E{-3}\U{A}} \\
78   \frac{6.00\U{V}}{1.20\E{-3}\U{A}} \\
79   \frac{6.00\U{V}}{2.00\E{-3}\U{A}}
80  \end{pmatrix}
81   &=
82  \begin{pmatrix}
83   1 & 1 & 1 \\
84   1 & \frac{1}{2} & 1 \\
85   1 & 1 & 0
86  \end{pmatrix}
87  \begin{pmatrix}
88   R_1 \\
89   R_2 \\
90   R_3
91  \end{pmatrix} \\
92  \begin{pmatrix}
93   R_1 \\
94   R_2 \\
95   R_3
96  \end{pmatrix}
97   =&
98  \begin{pmatrix}
99   1 & 1 & 1 \\
100   1 & \frac{1}{2} & 1 \\
101   1 & 1 & 0
102  \end{pmatrix}^{-1}
103  \begin{pmatrix}
104   6.00\U{k\Ohm} \\
105   5.00\U{k\Ohm} \\
106   3.00\U{k\Ohm}
107  \end{pmatrix}
108   =
109  \ans{
110   \begin{pmatrix}
111     1.00 \\
112     2.00 \\
113     3.00
114   \end{pmatrix}
115   \U{k\Ohm}
116  } \;.
117 \end{align}
118 \end{solution}